PT8.S01.Q10 - If the public library shared by adjacent towns..

grayaal28grayaal28 Core Member
edited January 2022 in Logical Reasoning 7 karma

Could you please explain why E is wrong?

Admin Note: Edited title. Please use the format: "PT#.S#.Q# - brief description of the question"

Comments

  • Scott MilamScott Milam Member Administrator Moderator Sage 7Sage Tutor
    1337 karma

    I think you mean PT 8, Section 1, Question 10. If not, please let me know!

    The key to any Strengthening question is to accurately identify the conclusion. In this case, the conclusion is: "A library in central Glenwood would be within walking distance for more people than would a library in central Redville."

    (E) is wrong because the fact that people are currently willing to walk "further than walking distance" to get to the current Redville library is irrelevant to the conclusion. Who cares whether people walk further than "walking distance?" All we care about is whether more people will live WITHIN walking distance.

    Put another way, the fact that people "walk further than walking distance" doesn't change the definition of "walking distance." It's as if the conclusion stated "more people would live within 1 mile" and answer (E) says "people are willing to walk more than 1 mile." That's great, but it doesn't really address the argument in any meaningful way.

    (B) is a better answer because it addresses a potential weakness of the argument - namely that even though more people live in central Glenwood, it is still possible for fewer people to be within walking distance if central Glenwood is much less dense. By clarifying that the two city centers are of the same size, it is much more likely that the conclusion is accurate, since both are the same size and central Glenwood is more populous.

    Hope that helps! Let me know if that doesn't make sense!

  • grayaal28grayaal28 Core Member
    7 karma

    Thank you.

Sign In or Register to comment.